23
$\begingroup$

In group theory the number of Sylow $p$-subgroups of a finite group $G$, is of the form $kp+1$.

So it is interesting to discuss about the divisors of this form. As I checked it seems that for an odd prime $p$, there is not any divisor $a$ of $p^4+1$, where $1<a<p^4+1$ and $a=kp+1$, for some $k>0$. Could you help me about this question? If it is true how we can prove it?

Also when I checked the same fact for $p^8+1$, we get many counterexamples. What is the difference between $4$ and $8$?

$\endgroup$
6
  • $\begingroup$ You checked for $p^4+1$ until which bound? $\endgroup$
    – YCor
    Jun 7, 2015 at 10:10
  • 1
    $\begingroup$ for primes $p$ less than 20000 by GAP $\endgroup$
    – BHZ
    Jun 7, 2015 at 10:15
  • 2
    $\begingroup$ Now YCor has settled this, of course. Note, however, that we have the easy factorization $p^{4}+1 = (p^{2} - \sqrt{2}p +1)(p^{2}+\sqrt{2}p +1)$ in the Euclidean ring $\mathbb{Z}[\sqrt{2}]$ (both factors clearly congruent to $1$ (mod $p$), though $p$ need not be prime in $\mathbb{Z}[\sqrt{2}]$). This is not especially relevant to counting Sylow $p$-subgroups in groups of course! $\endgroup$ Jun 8, 2015 at 8:25
  • 2
    $\begingroup$ Yes, groups of order $p^{i}(p^{4}+1)$ are interesting by virtue of the answer to the question. $\endgroup$ Jun 8, 2015 at 10:38
  • 2
    $\begingroup$ Indeed, it implies that a group $G$ of order $p^i(p^4+1)$ has characteristic subgroups $N\subset M$ such that $G/M$ and $N$ are $p$-groups, and $M/N$ has order $p^4+1$. $\endgroup$
    – YCor
    Jun 12, 2015 at 16:23

2 Answers 2

37
$\begingroup$

There's none indeed.

Lemma: if $1<m<n$ are coprime integers then $mn+1$ does not divide $n^4+1$.

First observe that for any $m,n$, of $mn+1$ divides $n^4+1$, then it divides $n^4m^4+m^4=((nm)^4-1)+1+m^4$, and since $mn+1$ clearly divides $((nm)^4-1)$, we deduce that it also divides $1+m^4$.

To prove the lemma, assume the contrary. As we have just seen, $mn+1$ divides $m^4+1$ as well. Write $(m^4+1)/(mn+1)=k$, and $k=(\ell m+r)$ with $0\le r\le m-1$. Then $m^4+1=(\ell m+r)((mn+1)=mN'+r$, so $r=1$. So $(\ell m+1)$ divides $m^4+1$. Then $m$ and $\ell$ are coprime: indeed, we have $(m\ell +1)(mn+1)=m^4+1$, so $\ell(mn+1)-m^3=-n$. If a prime $p$ were dividing both $m$ and $\ell$ then it would also divide $n$, contradicting that $m$ and $n$ are coprime, so $m$ and $\ell$ are indeed coprime. We have $\ell<n$ because otherwise $$m^4+1=(mn+1)(m\ell+1)\ge (mn+1)^2> (m^2+1)^2>m^4+1.$$ So we found a new pair $(m,\ell)$ with $\max(m,\ell)<n$, with $m\ell+1$ dividing both $m^4+1$ and $\ell^4+1$. So, assuming that $n$ is minimal, we're done unless $\ell=1$. This happens if $m+1$ divides $m^4+1$, and since $m+1$ divides $m^4-1$ as well, if this occurs then $m+1=2$, contradicting $m>1$.

(Note: without the coprime assumption the conclusion fails, as $(m,n)=(m,m^3)$ for $m\ge 2$, e.g. $(m,n)=(2,8)$, satisfies $mn+1|n^4+1$.)

Proposition If $p$ is prime then $p^4+1$ has no divisor of the form $kp+1$ except $1$ and $p^4+1$.

Proof: write $p^4+1=(kp+1)(k'p+\ell)$ with $0\le\ell\le p-1$; then $\ell=1$. So exchanging $k$ and $k'$ if necessary we can suppose $k\le k'$. If by contradiction $k$ is divisible by $p$ then $k'\ge k\ge p$, so $p^4+1\ge (p^2+1)^2>p^4+1$, contradiction. So $k$ and $p$ are coprime, and the lemma yields a contradiction.

$\endgroup$
11
  • $\begingroup$ The proof is very nice, thanks YCor. $\endgroup$
    – BHZ
    Jun 7, 2015 at 11:35
  • 1
    $\begingroup$ when we get that any integer less than $p$ is coprime with respect to it (in the last line of the proof). $\endgroup$
    – BHZ
    Jun 7, 2015 at 11:46
  • 16
    $\begingroup$ A great example of Vieta jumping! en.wikipedia.org/wiki/Vieta_jumping [This technique has been burned into my brain since 1988, when I failed to solve an IMO problem requiring this trick.] $\endgroup$
    – Terry Tao
    Jun 7, 2015 at 15:23
  • 15
    $\begingroup$ Actually, now that I think about it, the problem is in fact closely related to IMO 1988 Q6, since if $kp+1$ divides $p^4+1$, then it also divides $k^2+p^2$. $\endgroup$
    – Terry Tao
    Jun 7, 2015 at 15:33
  • 1
    $\begingroup$ @crskhr If it divides $p^4+1$ then it also divides $p^4+1-(kp+1)=p(p^3-k)$. Since $kp+1$ is coprime to $p$ this implies $kp+1$ divides $p^3-k$, hence divides $kp^3-k^2$. Hence it divides $-(kp^3-k^2)+p^2(kp+1)=k^2+p^2$. $\endgroup$
    – YCor
    Sep 29, 2019 at 10:13
2
$\begingroup$

Let $p$ be prime. Assume that $q \mid p^4+1$, where $q = ap+1$ for some $a \in \mathbb{N}$.

Then $(p^4+1)/q \equiv 1 \!\! \mod p$, hence $p^4+1 = (ap+1)(bp+1)$ for some $b \in \mathbb{N}$. Now we have $p^4 = abp^2+(a+b)p$, respectively, $p^3-abp-a-b = 0$. Therefore, effectively we are looking for solutions to the diophantine equation $x^3-xyz-y-z=0$ in positive $x$, $y$, $z$, and prime $x$.

When allowing negative (or zero) $x, y, z$, we would get a lot of solutions, and for positive $x, y, z \leq 1000$, we still find (up to switching of $y$ and $z$) $$ (x,y,z) \in \{(8,30,2),(27,240,3),(30,112,8),(112,418,30)\}. $$ I don't know whether there is a solution with prime $x$ as required -- however we have a diophantine equation of total degree $3$ in $3$ variables, and such are (at least potentially) hard.

$\endgroup$

Your Answer

By clicking “Post Your Answer”, you agree to our terms of service and acknowledge you have read our privacy policy.

Not the answer you're looking for? Browse other questions tagged or ask your own question.